Aller au contenu

onde EM à polarisation circulaire mettrait-elle un aimant en rotation ?


Paul_Wi11iams

Messages recommandés

Si l'un des physiciens du forum pouvait jeter un coup d’œil critique et bienveillant sur ma réflexion suivante qui part d'un post de l'autre jour.

...émission LW à polarisation circulaire. Mise en rotation [d'un aimant] avec axe radiale à un émetteur terrestre[/b].

 

Je raisonne en termes de l’électromagnétisme de base tel que ça s'applique depuis Marconi.

 

Des expériences et applications* montrent qu'il est possible de faire tourner un aimant à 1 000 000 RPM = 16 000 RPS en tours par seconde sans le faire éclater.

 

Or les ondes radio longues "LW" fonctionnent dans la bande 3khz à 300khz.

 

Si on considère un aimant placé transversalement à un faisceau LW à polarisation circulaire de 16kHz, le vecteur magnétique aurait-il tendance à mettre l'aimant en rotation ?

 

Selon le principe de réversibilité, on pourrait ensuite éteindre le faisceau radio et cet aimant en rotation deviendrait émetteur à son tour.

En accord avec les principes de conservation, la radiation électromagnétique émise devrait freiner sa rotation progressivement.

 

S'il n'y avait pas une faille, on aurait déjà trouvé des applications pour l'idée.

Mais la faille est où justement ?

 

* Réfs:

Moteur à 1M RPM = celeroton.com/en/products.html

Aimant en rotation (ne pas imiter) =

Modifié par Paul_Wi11iams
Lien vers le commentaire
Partager sur d’autres sites

Posté (modifié)

En attendant une réponse ;)* je pensais revoir ma suggestion en comparant avec le démarrage d'un moteur (tri)phasé à induction. On voit que le rotor n'a pas vocation à être un aimant permanent car dans ce cas le démarrage ne s'effectuerait pas correctement: Moteur à l'arrêt, le champ en rotation passerait trop vite devant les pôles de l'amant pour que l'effort de rotation soit sensible.

 

Du coup, je modifie la proposition et considère un "rotor" fait d'une bille en fer dont tout champ permanent ne participe pas au démarrage. A chaque instant, le champ magnétique en rotation induit un courant dans le rotor qui génère un champ magnétique de polarité N-S opposé, et en retard sur celui-ci. Ce champ tente de rattraper le champ en rotation et entraîne la bille à tourner.

 

Par contre, la bille aurait besoin d'être aimantée pour la permettre de réémettre en tant que transpondeur.

 

On remarquera que la réémission baisserait en fréquence selon une courbe caractéristique ou "signature" dépendant de la masse et forme de la bille. Mais là, on est déjà dans les applications. Or dans ce fil je voulais d'abord valider/invalider le principe.

 

* normalement je ne poste dans la rubrique "sciences" que quant il fait beau sur l'Europe car au regard du ciel, c'est bien le fil "temps de m..." qui passera en tête de file. D'après la météo ce n'est pas près de s'arranger !

 

L=53

Modifié par Paul_Wi11iams
Lien vers le commentaire
Partager sur d’autres sites

  • 2 semaines plus tard...
Posté (modifié)

Après avoir demandé un peu autour, je m'aperçois que même les experts ont du mal avec ma question.

Du coup, je la reformule et vais poster direct sur Physicis StackExchange:

 

avant traduction:

Une onde à polarisation circulaire transfère-t-elle un moment angulaire de l'émetteur à un récepteur ?

 

Comme base de comparaison, on considère un moteur à induction. Le stator induit un champ magnétique au rotor. Le champ au rotor suit l'aimantage successif des pôles du stator et entraîne sa rotation.

 

Dans le cas présent, nous remplaçons le champ du stator par une onde électromagnétique à polarisation circulaire. Nous remplaçons le rotor avec une petite bille ferreuse en apesanteur. Nous considérons le vecteur magnétique qui a l'apparence d'être en rotation.

 

Un champ magnétique serait induit dans la bille. Ce champ tendrait à suivre la rotation du vecteur magnétique de l'onde et devrait mettre la bille en rotation.

 

Par exemple, une onde LW de 16 kHz enclencherait une vitesse de rotation de 960 000 rpm, vitesse atteinte par certains moteurs sur le marché.

 

Suivant le principe de conservation de moment angulaire, l’émetteur devrait subir un effort de rotation inverse.

 

Selon le principe de réversibilité: Si on éteint l'émetteur, la bille magnétisée en rotation devrait devenir émetteur.

 

Si c'était le cas, on aurait déjà des applications (telles que la stabilisation des sondes spatiales ou des transpondeurs sans composant actif).

Ce n'est pas le cas. Pourquoi?

 

après traduction:

This question is about the rotation of macroscopic objects and looks at the magnetic vector of an electromagnetic wave.

 

As basis for comparison, we consider an induction motor. The stator induces a magnetic field in the rotor. The field in the rotor follows the magnetization of successive stator poles and causes its rotation.

 

In this question, we replace the stator field by a circularly polarized electromagnetic wave. We replace the rotor with a small iron sphere (ball-bearing or similar) in weightless in empty space. We consider the magnetic vector that appears to be rotating.

 

This should induce a current and then a magnetic field in the sphere. This field would tend to follow the rotation of the magnetic vector of the wave and put the ball in rotation. Any momentum not captured would continue to be later intercepted by other conducting objects or even never.

 

For example, a LW transmission at 16 kHz would trigger a rotational speed of 960 000 rpm, speed attained by some electric motors.

 

According the law of conservation of angular momentum, the transmitter should undergo an equal and opposite momentum change.

 

According to the principle of reversibility: If you turn off the transmitter, the rotating magnetized sphere should behave as a transmitter.

 

If all this were to be true, there should be existing applications such as stabilization of spacecraft or transponders without active components.

 

This does not seem to be the case.

 

Why not?

 

Edit, following comment: There may be a SETI application in which an object on the scale of one gramme would need to become detectable over long periods. nature.com/SETI-detection-by-data-carrying-objects Thus it could spin up reacting to natural radio background over a timescale of 10^8 years. If superconducting, magnetized and rotating near 1 000 000 RPM, would it generate a significant radio signal detectable from short distances ?

 

Je vais d'abord poster sur StackExchange.

Puis je donnerai les nouvelles ici.

Modifié par Paul_Wi11iams
Lien vers le commentaire
Partager sur d’autres sites

Posté (modifié)

lien vers le topic

physics.stackexchange.com/questions/262399/does-a-circularly-polarized-electromagnetic-wave-transfer-angular-momentum-from-a-transmitter

 

Les réponses étaient moins mathématiques que je ne craignais. Pour le principe, j'avais bien compris pour la transmission du couple de rotation, mais la quantité de mouvement transmise serait insuffisant pour des applications spatiales.

 

Ceci dit, je vais prendre le temps pour tester la formule qu'on me donne ci après, et ceci pour voir la puissance d'émission nécessaire pour empêcher la mise en rotation d'une sonde: Une puissance demandée doit être en proportion avec la vitesse d'augmentation de rotation parasite.

Autrement dit, cela correspond à la consommation des rétrofusées d'orientation divisée par la durée d'une mission.

 

Pour l'application SETI, se serait plus compliqué à évaluer.

 

En tout cas, les participants m'ont accordé deux "points" pour ma question, ce qui veut dire qu'ils l'ont trouvé sérieuse - et le niveau de ces gens est égal au meilleurs sur WA. Les moins bonnes questions sur PhysicsStackExchange sont simplement fermées par les participants/modérateurs.

Comme les réponses sont accessibles sur leur site, je n'ai pas pris le temps pour les démêler. Comme la question n’ameute pas les foules ici, je n'ai pas pris le temps pour traduire. Ceci dit, j'ai eu un mot encourageant de la part d'un webastram "pro", ce qui m'a donné confiance pour avancer. Il pensait que le problème tiendrait davantage aux ordres de grandeur qu'aux principes mis en œuvre, ce qui s'est révélé juste.

up voted

 

 

Yes, there is an angular momentum transfer, but it is tiny and very hard to measure (which makes it technologically irrelevant). The speed of rotation would not be determined by the frequency of the wave but by the friction and moments of inertia of the system that experiences the torque, so it's unlikely that anything but atomic and molecular systems can be "spun up" to the classical rotation frequencies. – CuriousOne 22 hours ago

 

 

thanks. One application would be SETI inscribed matter <nature.com/nature/journal/v431/n7004/full/nature02884.html>;. example: one-gramme mass free-floating in space to be spun-up over 10^8 years. I Will edit my question. – paul_wi11iams 22 hours ago

 

 

 

@CuriousOne You can spin up a bunch of particles that are at least microscopic (i.e. ~1 micron) in size. I'm not sure it goes as far as "anything you can put in optical tweezers", but it's definitely way past atomic and molecular systems. – Emilio Pisanty 21 hours ago

 

 

 

@EmilioPisanty: Sorry for the misunderstanding, I was talking about reaching the frequency of the light. Optical tweezers aren't spinning things up to 1015Hz

. I think molecular rotation is as far as one can get, and that's in the far IR, if I remember correctly. I agree, one can, of course, move things that are much larger, just not as fast as the OP has in mind. It's an interesting question if e.g. an electric motor can be thought of as transferring macroscopic angular momentum trough the near field of the electromagnets? – CuriousOne 21 hours ago

 

 

 

@CuriousOne Oh, in that case, then yes. – Emilio Pisanty 16 hours ago

 

add a comment | show 2 more comments

1 Answer

active

oldest

votes

up vote

3

down vote

accept

 

 

All of your claims are essentially true. The angular momentum of light, in both its orbital and spin varieties, is indeed angular momentum that can be transferred to matter to make it spin and give it the garden variety of mechanical angular momentum. This is well explained in the relevant Wikipedia section, with good references for experiments that show it. If you want something more tangible, this video is a good starting point, showing particles spinning under the action of circularly polarized light.

 

Where you fail, however, is in your estimation:

 

If all this were to be true, there should be existing applications such as stabilization of spacecraft or transponders without active components.

 

The effect is definitely present, but that doesn't mean that it's present in a sufficiently strong capacity to be technologically useful. In particular, each photon of light carries a spin angular momentum of ℏ

, and an energy of ℏω, and from here it follows that the ratio of torque to power carried by a monochromatic beam is fixed at

torquepower=SAM/Δtenergy/Δt=ℏℏω=1ω.

If you have a wave at ν=16kHz, that means that for every watt of power you get a torque of

τ=Pω=1W2π×16kHz≈9.95×10−6Nm.

 

Or put another way, if you want to stabilize a spacecraft, using torques of several hundred newton meters, you'd have to arrange for the spacecraft to interact with (i.e. absorb, or ideally completely reflect, though of course even a 99% reflectance will mean a lot of absorbed power) something like a megawatt of power in the beam. Whilst possible in principle, this is not a reasonable technological solution compared to all the other ways to impart torque to objects.

shareciteeditflag

 

answered 21 hours ago

Emilio Pisanty

36.8k580180

 

 

 

I didn"t make it clear that the transmitter would be onboard the spacecraft (using counter-rotation)and the power needed would be proportional to speed of acccumulation of rotation to be countered. I will take my time to use your formuea as I'm not in physics. I will also work through the first part of your answer for the SETI application that I edited into my question to answer the comment from CuriousOne. Thanks for now. – paul_wi11iams 19 hours ago

upvote

flag

 

Either way, the torque to power ratio is simply too small to be useful. You've got a MW power source in board? There's a gazillion better ways to use it. "Proportional" only works when the constant of proportionality doesn't have six more zeros than you need. – Emilio Pisanty 16 hours ago

Modifié par Paul_Wi11iams
Lien vers le commentaire
Partager sur d’autres sites

Posté (modifié)
Waouhhh bonne question ?!

 

Il y a intérêt.

 

Sur PhysicsStackExchange, si on ne comprend pas de quoi on parle, surtout sur un sujet "compliqué", ils ne font pas de cadeau. Il faut voir le pédigrée (MIT, Caltech...)

 

Merci pour ta réponse.

Si tu peux développer, je veux bien !

 

L=225

Modifié par Paul_Wi11iams
Lien vers le commentaire
Partager sur d’autres sites

Je pense que ça marche pas. Un aimant est généralement conducteur est donc l'onde EM ne peut pas se propager en son sain (loi de maxwell), et l'effet de peau n'est pas suffisant à mon humble avis, faut que je ressorte mes cours d'eletromag. La question est vraiment interressant, ce que je peut dire c'est que plus la fréquence est élevée moins le champ pénétre...

Je vais étudié le truc ça m'intéresse.

Comme il est écrit plus on travail à "basse fréquence". La conductivité/perméabilité magnétique de l'aimant sont aussi des paramètres importants il permette de quantifié la profondeur de penetratiin de l'em dans le matériaux, est donc du volume de matière qui sera influencé par l'em... C'est pas simple, mais j'adore ce genre de probleme.

Merci pour ta question. Pour une fois je vais essayer de ne pas être trop feignant et bosser le sujet.

Modifié par AstroFilDu76
Lien vers le commentaire
Partager sur d’autres sites

Posté (modifié)
Je pense que ça marche pas. Un aimant est généralement conducteur est donc l'onde EM ne peut pas se propager en son sain (loi de maxwell), et l'effet de peau n'est pas suffisant à mon humble avis, faut que je ressorte mes cours d'eletromag. La question est vraiment interressant, ce que je peut dire c'est que plus la fréquence est élevée moins le champ pénétre...

Je vais étudié le truc ça m'intéresse.

Comme il est écrit plus on travail à "basse fréquence". La conductivité/perméabilité magnétique de l'aimant sont aussi des paramètres importants il permette de quantifié la profondeur de penetratiin de l'em dans le matériaux, est donc du volume de matière qui sera influencé par l'em... C'est pas simple, mais j'adore ce genre de probleme.

Merci pour ta question. Pour une fois je vais essayer de ne pas être trop feignant et bosser le sujet.

Je t'en prie.

C'est déjà sympa de m'avoir répondu !

 

Clarification:

J'essaie de re-situer ma question parce que je parle bien de la détection d'une onde, pas de la faire pénétrer dans une objet conducteur:

  1. Une antenne (qui peut être suivi d'un diode) constitue un système de détection qui transforme l'énergie de l'onde EM en mouvement d'électrons dans une paire de fils.
  2. Dans le cas présent, on constitue un système de détection qui fait mouvoir des atomes entiers.

Dans les deux cas, l'onde EM disparaît, en passant par une interface, pour être remplacée par autre chose.

Le rôle de la bille ferreux n'est pas de laisser entrer une onde mais d'interagir avec.

 

Réversibilité.

Dans le premier cas l'action est réversible comme pour un réflecteur de radar sur un bateau de pêche. Sans aucun composant électronique, il se contente de traiter l'onde et la renvoyer dans sa direction d'arrivée.

 

Dans le deuxième cas, j'envisage que la bille en rotation se comporterait en transmetteur.

 

Je prends acte de la critique:

La réponse à ma première question sur StackExchange montrait que le couplage de l'onde EM à polarisation circulaire était trop faible pour que la bille fonctionne en récepteur.

 

La question modifiée pour en tenir compte.

Maintenant, et ici la question change, considérons un aimant permanent situé en apesanteur dans l'espace. Nous le mettons en rotation avec son champ Nord/Sud qui tourne sur son centre longitudinal.

 

Le champ peut être très puissant car l'aimant peut être une boucle supraconducteur.

 

Dans un premier temps, on imagine cet aimant à l'intérieur d'une boucle de cuivre fixe. Le tout constitue un alternateur et à 16000 tour/sec, la boucle externe serait aussi un émetteur radio.

Maintenant, enlevons la boucle cuivre: il y a néanmoins une tension électrique dans l'espace qu'elle occupait.

 

Dans ce cas est-ce qu l'émission radio continue ?

Modifié par Paul_Wi11iams
Lien vers le commentaire
Partager sur d’autres sites

Rejoignez la conversation !

Vous pouvez répondre maintenant et vous inscrire plus tard. Si vous avez un compte, connectez-vous pour poster avec votre compte.

Invité
Répondre à ce sujet…

×   Collé en tant que texte enrichi.   Coller en tant que texte brut à la place

  Seulement 75 émoticônes maximum sont autorisées.

×   Votre lien a été automatiquement intégré.   Afficher plutôt comme un lien

×   Votre contenu précédent a été rétabli.   Vider l’éditeur

×   Vous ne pouvez pas directement coller des images. Envoyez-les depuis votre ordinateur ou insérez-les depuis une URL.

  • En ligne récemment   0 membre est en ligne

    • Aucun utilisateur enregistré regarde cette page.
×
×
  • Créer...

Information importante

Nous avons placé des cookies sur votre appareil pour aider à améliorer ce site. Vous pouvez choisir d’ajuster vos paramètres de cookie, sinon nous supposerons que vous êtes d’accord pour continuer.